0% found this document useful (0 votes)
215 views39 pages

2021 AMC 10B Solution

美国数学竞赛2021 年 AMC10 B卷的题解,含多种解法以及讲解

Uploaded by

panda.yh0207
Copyright
© © All Rights Reserved
We take content rights seriously. If you suspect this is your content, claim it here.
Available Formats
Download as PDF, TXT or read online on Scribd
0% found this document useful (0 votes)
215 views39 pages

2021 AMC 10B Solution

美国数学竞赛2021 年 AMC10 B卷的题解,含多种解法以及讲解

Uploaded by

panda.yh0207
Copyright
© © All Rights Reserved
We take content rights seriously. If you suspect this is your content, claim it here.
Available Formats
Download as PDF, TXT or read online on Scribd
You are on page 1/ 39

2021 AMC 10B Solution

Problem1

How many integer values of satisfy ?

Solution 1
Since is about , we multiply 9 by 2 for the numbers from to and the

numbers from to and add 1 to account for the zero to get


~smarty101 and edited by Tony_Li2007

Solution 2
There are two cases here.

When and So then

When and So then . Dividing


by and flipping the sign, we get
From case 1 and 2, we know that . Since is an integer,
we must have between and . There are a total

of

Problem2

What is the value of

Solution
Note that the square root of any square is always the absolute value of the
squared number because the square root function will only return a positive
number. By squaring both and , we see that ,

thus is negative, so we must take the absolute value

of , which is just . Knowing this, the first term in the

expression equals and the second term is , and

summing the two gives .

~bjc, abhinavg0627 and JackBocresion

Solution 2

Let ,

then .

The term is there due to difference of squares. Simplifying the

expression gives us , so ~ shrungpatel

Problem3
In an after-school program for juniors and seniors, there is a debate team with an
equal number of students from each class on the team. Among the students

in the program, of the juniors and of the seniors are on the debate
team. How many juniors are in the program?
Solution 1

Say there are juniors and seniors in the program. Converting percentages to

fractions, and are on the debate team, and since an equal number of

juniors and seniors are on the debate team,

Cross-multiplying and simplifying we get Additionally, since there

are students in the program, It is now a matter of solving the

system of equations and the solution

is Since we want the number of juniors, the answer

is

Solution 2 (Fast and not rigorous)

We immediately see that is the only possible amount of seniors, as can


only correspond with an answer choice ending with . Thus the number of

seniors is and the number of juniors is .

Solution 3
Since there are an equal number of juniors and seniors on the debate team,
suppose there are juniors and seniors. This number

represents of the juniors and of the seniors, which


tells us that there are juniors and seniors. There are juniors and

seniors in the program altogether, so we get


Which means there are juniors on the debate

team, .

Problem4
At a math contest, students are wearing blue shirts, and another students
are wearing yellow shirts. The 132 students are assigned into pairs. In
exactly of these pairs, both students are wearing blue shirts. In how many
pairs are both students wearing yellow shirts?

Solution
There are students paired with a blue partner. The other students
wearing blue shirts must each be paired with a partner wearing a shirt of the
opposite color. There are students remaining. Therefore the requested

number of pairs is

Problem5
The ages of Jonie's four cousins are distinct single-digit positive integers. Two of
the cousins' ages multiplied together give , while the other two multiply to .
What is the sum of the ages of Jonie's four cousins?

Solution
First look at the two cousins' ages that multiply to . Since the ages must be
single-digit, the ages must either be or
Next, look at the two cousins' ages that multiply to . Since the ages must be
single-digit, the only ages that work are Remembering that all the
ages must all be distinct, the only solution that works is when the ages

are and .

We are required to find the sum of the ages, which

is
Problem6
Ms. Blackwell gives an exam to two classes. The mean of the scores of the
students in the morning class is , and the afternoon class's mean score is .
The ratio of the number of students in the morning class to the number of

students in the afternoon class is . What is the mean of the scores of all the
students?

Solution 1
WLOG, assume there are students in the morning class and in the afternoon

class. Then the average is

Solution 2
Let there be students in the morning class and students in the afternoon

class. The total number of students is . The average

is . Therefore, the answer is .

Problem7

In a plane, four circles with radii and are tangent to line at the same

point but they may be on either side of . Region consists of all the points
that lie inside exactly one of the four circles. What is the maximum possible area
of region ?
Solution

After a bit of wishful thinking and


inspection, we find that the above configuration maximizes our area, which

is

Problem8
Mr. Zhou places all the integers from to into a by grid. He
places in the middle square (eighth row and eighth column) and places other
numbers one by one clockwise, as shown in part in the diagram below. What is
the sum of the greatest number and the least number that appear in the second

row from the top?


Solution 1
By observing that the right-top corner of a square will always be a square, we
know that the top right corner of the x grid is . We can subtract to
get the value of the top-left corner; . We can then find the value of the
bottom left and right corners similarly. From there, we can find the value of the
box on the far right in the 2nd row from the top by subtracting , since the
length of the side will be one box shorter. Similarly, we find the value for the box
2nd from the left and 2nd from the top, which is . We know that the least
number in the 2nd row will be , and the greatest will be the number to its left,

which is less than . We then sum and to get .

-Dynosol

Solution 2: Draw It Out

Drawing out the diagram, we get . Note that this should mainly be
used just to check your answer.

Problem9

The point in the -plane is first rotated counterclockwise

by around the point and then reflected about the line .

The image of after these two transformations is at . What


is

Solution

The final image of is . We know the reflection rule for reflecting

over is . So before the reflection

and after rotation the point is .


By definition of rotation, the slope between and must be

perpendicular to the slope between and . The first slope

is . This means the slope of and is .


Rotations also preserve distance to the center of rotation, and since we only

"travelled" up and down by the slope once to get from to it

follows we shall only use the slope once to travel from to .

Therefore point is located at . The answer

is .

Problem10
An inverted cone with base radius and height is full of water.
The water is poured into a tall cylinder whose horizontal base has radius
of . What is the height in centimeters of the water in the cylinder?

Solution 1

The volume of a cone is where is the base radius and is


the height. The water completely fills up the cone so the volume of the water

is .

The volume of a cylinder is so the volume of the water in the


cylinder would be .
We can equate these two expressions because the water volume stays the same

like this . We get and .


So the answer is

Solution 2 (ratios)
The water completely fills up the cone. For now, assume the radius of both cone

and cylinder are the same. Then the cone has of the volume of the cylinder,
and so the height is divided by . Then, from the problem statement, the radius is

doubled, meaning the area of the base is quadrupled (since ).

Therefore, the height is divided by and divided by , which

is

Problem11
Grandma has just finished baking a large rectangular pan of brownies. She is
planning to make rectangular pieces of equal size and shape, with straight cuts
parallel to the sides of the pan. Each cut must be made entirely across the pan.
Grandma wants to make the same number of interior pieces as pieces along the
perimeter of the pan. What is the greatest possible number of brownies she can
produce?

Solution 1
Let the side lengths of the rectangular pan be and . It follows

that , since half of the brownie pieces are in the


interior. This
gives

.
Adding 8 to both sides and applying Simon's Favorite Factoring Trick, we

obtain . Since and are both positive, we

obtain (up to ordering). By

inspection, maximizes the number of brownies.


Solution 2
Obviously, no side of the rectangular pan can have less than brownies beside
it. We let one side of the pan have brownies, and let the number of brownies on

its adjacent side be . Therefore, , and solving


yields and there are brownies in the pan. is the
only choice larger than , but it cannot be the answer since the only way to
fit brownies in a pan without letting a side of it have less than brownies
beside it is by forming a square of brownies on each side, which does not meet

the requirement. Thus the answer is .

Problem12
Let . What is the ratio of the sum of the odd
divisors of to the sum of the even divisors of ?

Solution 1

Prime factorize to get . For each odd

divisor of , there exist even divisors of , therefore the

ratio is

Solution 2

Prime factorizing , we see . The sum of 's


odd divisors are the sum of the factors of without , and the sum of the even
divisors is the sum of the odds subtracted by the total sum of divisors. The sum of
odd divisors is given
by

and the total sum of divisors


is
. Thus, our ratio is

Problem13
Let be a positive integer and be a digit such that the value of the

numeral in base equals , and the value of the numeral in

base equals the value of the numeral in base six. What is

Solution

We can start by setting up an equation to convert base to base 10. To

convert this to base 10, it would be Because it is equal to


263, we can set this equation to 263. Finally, subtract from both sides to

get .

We can also set up equations to convert base and base 6 to

base 10. The equation to covert base to base 10

is The equation to convert base 6 to base 10

is

Simplify so it becomes Setting the above

equations equal to each other, we have

Subtracting 4 from both sides gets


We can then use equations

to solve for . Set equal

to and solve to find that .

Plug back into the equation . Subtract 261

from both sides to get your final equation of Solve


using the quadratic formula to find that the solutions are 9 and -10. Because the
base must be positive,

Adding 2 to 9 gets

-Zeusthemoose (edited for readability)

Solution 2
is greater than when both are interpreted in base 10, so is less
than . Some trial and error gives . in base 9 is , so the

answer is .

Problem14
Three equally spaced parallel lines intersect a circle, creating three chords of

lengths and . What is the distance between two adjacent parallel


lines?
Solution 1

Since two parallel chords have the same length ( ), they must be equidistant
from the center of the circle. Let the perpendicular distance of each chord from
the center of the circle be . Thus, the distance from the center of the circle to
the chord of length is

and the distance between each of the chords is just . Let the radius of the
circle be . Drawing radii to the points where the lines intersect the circle, we
create two different right triangles:

- One with base , height , and hypotenuse ( on the


diagram)

- Another with base , height , and hypotenuse ( on


the diagram)
By the Pythagorean theorem, we can create the following system of equations:
Solving, we find , so .

Solution 2 (Coordinates)
Because we know that the equation of a circle

is where the center of the circle

is and the radius is , we can find the equation of this circle by centering

it on the origin. Doing this, we get that the equation is . Now,


we can set the distance between the chords as so the distance from the
chord with length 38 to the diameter is .
Therefore, the following points are on the circle as the y-axis splits the chord in
half, that is where we get our x value:

Now, we can plug one of the first two value in as well as the last one to get the
following equations:

Subtracting these two equations, we get - therefore, we

get . We want to
find because that's the distance between two chords. So, our answer

is .
Problem15

The real number satisfies the equation . What is the value

of

Solution 1

We square to get . We subtract 2 on

both sides for and square again, and see

that so . We can divide our original

expression of by to get that it is equal

to . Therefore because is 7, it is equal

to .

Solution 2

Multiplying both sides by and using the quadratic formula, we get .

We can assume that it is , and notice that this is also a solution the

equation , i.e. we have . Repeatedly using


this on the given (you can also just note Fibonacci

numbers),

~Lcz

Solution 3

We can immediately note that the exponents of are an


arithmetic sequence, so they are symmetric around the middle term.

So, . We can see that

since , and

therefore . Continuing from here, we

get , so . We don't even need to

find what is! This is since is evidently , which is our answer.

Problem16
Call a positive integer an uphill integer if every digit is strictly greater than the

previous digit. For example, and are all uphill integers,


but and are not. How many uphill integers are divisible
by ?

Solution 1
The divisibility rule of is that the number must be congruent to mod and
congruent to mod . Being divisible by means that it must end with a or
a . We can rule out the case when the number ends with a immediately
because the only integer that is uphill and ends with a is which is not
positive. So now we know that the number ends with a . Looking at the answer
choices, the answer choices are all pretty small, so we can generate all of the
numbers that are uphill and are divisible by . These numbers

are which are numbers C.

Solution 2
First, note how the number must end in either or in order to satisfying being
divisible by . However, the number can't end in because it's not strictly
greater than the previous digits. Thus, our number must end in . We do
casework on the number of digits.
Case 1 = digit. No numbers work, so

Case 2 = digits. We have the numbers and , but isn't an uphill


number, so numbers.

Case 3 = digits. We have the numbers . So numbers.

Case 4 = digits. We have the numbers and , but


only satisfies this condition, so number.
Case 5= digits. We have only , so number.

Adding these up, we have .


~JustinLee2017
Solution 3
Like solution 2, we can proceed by using casework. A number is divisible by if

is divisible by and In this case, the units digit must be otherwise no


number can be formed.
Case 1: sum of digits = 6
There is only one number,
Case 2: sum of digits = 9
There are two numbers: and
Case 3: sum of digits = 12
There are two numbers: and
Case 4: sum of digits = 15
There is only one number,
We can see that we have exhausted all cases, because in order to have a larger
sum of digits, then a number greater than needs to be used, breaking the

conditions of the problem. The answer is

Problem17
Ravon, Oscar, Aditi, Tyrone, and Kim play a card game. Each person is given 2

cards out of a set of 10 cards numbered The score of a


player is the sum of the numbers of their cards. The scores of the players are as
follows: Ravon--11, Oscar--4, Aditi--7, Tyrone--16, Kim--17. Which of the
following statements is true?

Solution

Oscar must be given 3 and 1, so we rule out and . If Tyrone had card
7, then he would also have card 9, and then Kim must have 10 and 7 so we rule

out . If Aditi was given card 4, then she would have card 3, which Oscar

already had. So the answer is


~smarty101 and smartypantsno_3

Solution 2
Oscar must be given 3 and 1. Aditi cannot be given 3 or 1, so she must have 2
and 5. Similarly, Ravon cannot be given 1, 2, 3, or 5, so he must have 4 and 7,

and the answer is .

Problem18
A fair -sided die is repeatedly rolled until an odd number appears. What is the
probability that every even number appears at least once before the first
occurrence of an odd number?

Solution 1

There is a chance that the first number we choose is even.

There is a chance that the next number that is distinct from the first is even.

There is a chance that the next number distinct from the first two is even.

, so the answer is

~Tucker

Solution 2

Every set of three numbers chosen from has an equal


chance of being the first 3 distinct numbers rolled.
Therefore, the probability that the first 3 distinct numbers

are is

~kingofpineapplz

Solution 3 (Quicksolve)
Note that the problem is basically asking us to find the probability that in some

permutation of that we get the three even numbers in the first


three spots.

There are ways to order the numbers and ways to order the evens
in the first three spots and the odds in the next three spots.

Therefore the probability is .

--abhinavg0627

Solution 4

Let denote the probability that the first odd number appears on roll and all
our conditions are met. We now proceed with complementary counting.

For , it's impossible to have all evens appear before an odd. Note that

for

Summing for all , we get our answer


of

Problem19
Suppose that is a finite set of positive integers. If the greatest integer in is
removed from , then the average value (arithmetic mean) of the integers
remaining is . If the least integer in is also removed, then the average value
of the integers remaining is . If the greatest integer is then returned to the set,
the average value of the integers rises to The greatest integer in the original
set is greater than the least integer in . What is the average value of all
the integers in the set

Solution 1
Let be the greatest integer, be the smallest, be the sum of the numbers in
S excluding and , and be the number of elements in S.

Then,

Firstly, when the greatest integer is removed,

When the smallest integer is also removed,

When the greatest integer is added back,

We are given that

After you substitute , you have 3 equations with 3

unknowns , and .

This can be easily solved to yield , , .

average value of all integers in the set , D)


Solution 2
We should plug in and assume everything is true except the part. We
then calculate that part and end up with . We also see with the formulas
we used with the plug in that when you increase by the part

decreases by . The answer is then . You can work


backwards because it is multiple choice and you don't have to do critical thinking.

Problem20
The figure is constructed from line segments, each of which has length .

The area of pentagon can be written is ,

where and are positive integers. What is

Solution 1
Let be the midpoint of . Noting
that and are triangles because of the
equilateral
triangles,
. Also, and
so

Solution 2

Draw diagonals and to split the pentagon into three parts. We can
compute the area for each triangle and sum them up at the end. For

triangles and , they each have area .


For triangle , we can see

that and . Using Pythagorean Theorem,

the altitude for this triangle is , so the area is . Adding each part up,

we get .
Problem21

A square piece of paper has side length and vertices and in


that order. As shown in the figure, the paper is folded so that vertex meets

edge at point , and edge intersects edge at point .

Suppose that . What is the perimeter of triangle

Solution 1
We can set the point on where the fold occurs as point . Then, we can
set as , and as because of symmetry due to the fold. It
can be recognized that this is a right triangle, and solving for , we get,

We know this is a 3-4-5 triangle because the side lengths are . We

also know that is similar to because angle is a right angle.


Now, we can use similarity to find out that the perimeter is just the perimeter
of . Thats just . Therefore, the

final answer is
~Tony_Li2007

Solution 2
Let line we're reflecting over be , and let the points where it
hits and , be and , respectively. Notice, to reflect over a line
we find the perpendicular passing through the midpoint of the two points (which
are the reflected and the original). So, we first find the equation of the line . The

segment has slope , implying line has a slope

of . Also, the midpoint of segment is , so line passes


through this point. Then, we get the equation of line is

simply . Then, if the point where is reflected over

line is , then we get is the line . The intersection of

and segment is . So, we get .

Then, line segment has equation , so the point is

the -intercept, or . This implies that , and

by the Pythagorean Theorem, (or you could notice is

a right triangle). Then, the perimeter is , so

our answer is .
Solution 3 (Fakesolve):

Assume that E is the midpoint of . Then, and

since , . By the Pythagorean Theorem, .

It easily follows that our desired perimeter is ~samrocksnature

Problem22
Ang, Ben, and Jasmin each have blocks, colored red, blue, yellow, white, and
green; and there are empty boxes. Each of the people randomly and
independently of the other two people places one of their blocks into each box.
The probability that at least one box receives blocks all of the same color

is , where and are relatively prime positive integers. What is

Solution 1

Let our denominator be , so we consider all possible distributions.

We use PIE (Principle of Inclusion and Exclusion) to count the successful ones.
When we have at box with all balls the same color in that box, there

are ways for the distributions to occur ( for selecting

one of the five boxes for a uniform color, for choosing the color for that
box, for each of the three people to place their remaining items).
However, we overcounted those distributions where two boxes had uniform color,

and there are ways for the distributions to occur ( for

selecting two of the five boxes for a uniform color, for choosing the color for
those boxes, for each of the three people to place their remaining items).
Again, we need to re-add back in the distributions with three boxes of uniform
color... and so on so forth.
Our success by PIE
is

yielding an answer of .

Solution 2
As In Solution 1, the probability
is

Dividing by , we

get Dividing by , we

get Dividing by , we

get .

Solution 3

Use complementary counting. Denote as the total number of ways to

put colors to boxes by 3 people out of which ways are such that no box

has uniform color. Notice . From this setup we see the question is

asking for . To find we want to exclude the cases of a) one box


of the same color, b) 2 boxes of the same color, c) 3 boxes of same color, d) 4
boxes of the same color, and e) 5 boxes of the same color. Cases d) and e)
coincide. From this, we have
In case b), there are ways to choose 2 boxes that have the same

color, ways to choose the two colors, 2! ways to permute the 2 chosen

colors, and ways so that the remaining 3 boxes don’t have the same color.
The same goes for cases a) and c). In case e), the total number of ways to

permute 5 colors is . Now, we just need to calculate , and .

We have , since we subtract the


number of cases where the boxes contain uniform colors, which is 2.

In the same way, -


again, we must subtract the number of ways at least 1 box has uniform color.
There are ways if 3 boxes each contains uniform color. Two boxes each
contains uniform color is the same as previous. If one box has the same color,

there are ways to pick a box, and ways to pick a color for that box,

1! ways to permute the chosen color, and ways for the remaining 2 boxes to
have non-uniform colors.
Similarly,

Thus,
Thus, the probability is and the answer

is

Problem23
A square with side length is colored white except for black isosceles right
triangular regions with legs of length in each corner of the square and a black

diamond with side length in the center of the square, as shown in the
diagram. A circular coin with diameter is dropped onto the square and lands in
a random location where the coin is completely contained within the square. The
probability that the coin will cover part of the black region of the square can be

written as , where and are positive integers.

What is ?
Solution

To find the probability, we look at the . For the coin


to be completely contained within the square, we must have the distance from the

center of the coin to a side of the square to be at least , as it's the radius of the

coin. This implies the is a square with side

length , with an area of . Now, we consider cases on


where needs to land to partially cover a black region.
Near The Center Square

We can have the center of the coin land within of the center square, or inside
of the center square. We have that the center lands either outside of the square,

or inside. So, we have a region with emanating from every point on the exterior
of the square, forming quarter circles and rectangles. The quarter circles

combine to make a full circle, with radius of , so that has an area of . The

area of a rectangle is , so of them combine to an area

of . The area of the black square is simply . So, for this

case, we have a combined total of . Onto the second (and


last) case.
Near A Triangle

We can also have the coin land within of one of the triangles. By symmetry, we
can just find the successful region for one of them, then multiply by . Consider
this diagram. We can draw in an altitude from the bottom corner of the square to
hit the hypotenuse of the blue triangle. The length of this when passing through

the black region is , and when passing through the white region (while being

contained in the blue triangle) is . However, we have to subtract off when it


doesn't pass through the red square. Then, it's the hypotenuse of a small

isosceles right triangle with side lengths of , or . So, our altitude of the

blue triangle is . Then, recall, the area of an

isosceles right triangle is , where is the altitude from the right angle. So,

squaring this, we get . Now, we have to multiply this by to account

for all of the black triangles, to get as the final area for this case.

Finishing
Then, to have the coin touching a black region, we add up the area of our
successful regions,
or

. The total region is , so our probability

is , which

implies . This corresponds to answer

choice

Problem24
Arjun and Beth play a game in which they take turns removing one brick or two
adjacent bricks from one "wall" among a set of several walls of bricks, with gaps
possibly creating new walls. The walls are one brick tall. For example, a set of
walls of sizes and can be changed into any of the following by one

move: or

Arjun plays first, and the player who removes the last brick wins. For which
starting configuration is there a strategy that guarantees a win for Beth?

Solution
First we note that symmetrical positions are losing for the player to move. Then

we start checking small positions. is always winning for the first player.

Furthermore, is losing and so is We look at all the positions

created from as is obviously winning by

playing There are several different positions that can be played by

the first player from They


are

Now we list refutations for each of these moves:


This proves that is losing for the first player.

-Note: In general, this game is very complicated. For

example is winning for the first player but good luck showing
that.

Solution 2 (Process of Elimination)

can be turned into by Arjun, which is symmetric, so


Beth will lose.

can be turned into by Arjun, which is symmetric, so


Beth will lose.

can be turned into by Arjun, which is symmetric, so


Beth will lose.

can be turned into by Arjun, which is symmetric, so


Beth will lose.

That leaves or .

Solution 3 (Nim-values)
Let the nim-value of the ending game state, where someone has just removed
the final brick, be . Then, any game state with a nim-value of is losing. It is
well-known that the nim-value of a supergame (a combination of two or more
individual games) is the binary xor function on the nim-values of the individual
games that compose the supergame. Therefore, we calculate the nim-values of
the states with a single wall up to bricks long (since the answer choices only go
up to ).
First, the game with brick has a nim-value of .
Similarly, the game with bricks has a nim-value of .
Next, we consider a brick wall. After the next move, the possible resulting game
states are brick, a brick wall, or separate bricks. The first two options have

nim-values of and . The final option has a nim-value of , so the


nim-value of this game state is .
Next, the brick wall. The possible states are a brick wall, a brick wall,
a brick wall and a brick wall, or a brick wall and a brick wall. The nim-
values of these states are , , , and , respectively, and hence the nim-value
of this game state is .
[Why is the nim-value of it ? - awesomediabrine
https://en.wikipedia.org/wiki/Mex_(mathematics)]
The possible game states after the brick wall are the following: a brick wall,
a brick wall, a brick wall and a brick wall, a two brick walls, and a brick
wall plus a brick wall. The nim-values of these are , , , , and ,
respectively, meaning the nim-value of a brick wall is .
Finally, we find the nim-value of a brick wall. The possible states are a brick
wall, a brick wall and a brick wall, a brick wall and a brick wall, a brick
wall, a brick wall and a brick wall, and finally two brick walls. The nim-
values of these game states are , , , , , and , respectively. This means
the brick wall has a nim-value of .
The problem is asking which of the answer choices is losing, or has a nim-value

of . We see that option has a nim-value of ,

option has a nim-value of , option has a nim-

value of , option has a nim-value

of , and option has a nim-value of ,

so the answer is .

This method can also be extended to solve the note after the first solution. The
nim-values of the brick wall and the brick wall are and , using the same

method as above. The nim-value of is

therefore , which is winning.


Video Solution by OmegaLearn (Game Theory)
https://youtu.be/zkSBMVAfYLo

Problem25
Let be the set of lattice points in the coordinate plane, both of whose

coordinates are integers between and inclusive. Exactly points


in lie on or below a line with equation The possible values

of lie in an interval of length where and are relatively prime positive

integers. What is

Solution 1
First, we find a numerical representation for the number of lattice points in that
are under the line For any value of the highest lattice point

under is Because every lattice point

from to is under the line, the total number of lattice points

under the line is

Now, we proceed by finding lower and upper bounds for To find the lower
bound, we start with an approximation. If lattice points are below the line,

then around of the area formed by is under the line. By using the formula

for a triangle's area, we find that when Solving

for assuming that is a point on the line, we

get Plugging in to we get:


We have a repeat every values (every time goes through a lattice
point). Thus, we can use arithmetic sequences to calculate the value above:

This means that is a possible value of Furthermore, it is the lower bound

for This is because goes through many points (such

as ). If was lower, would no longer go through some of


these points, and there would be less than lattice points under it.
Now, we find an upper bound for Imagine increasing slowly and rotating

the line starting from the lower bound of The upper


bound for occurs when intersects a lattice point again (look at this
problem to get a better idea of what's
happening: https://artofproblemsolving.com/wiki/index.php/2011_AMC_10B_Prob
lems/Problem_24).

In other words, we are looking for the first that is expressible as a ratio

of positive integers with For each , the smallest


multiple of which

exceeds is respectively, and

the smallest of these is Note: start listing the multiples of from and

observe that they get further and further away from Alternatively, see the
method of finding upper bounds in solution 2.

The lower bound is and the upper bound is Their difference is so

the answer is

~JimY

Solution 2

I know that I want about of the box of integer coordinates above my line. There
are a total of 30 integer coordinates in the desired range for each axis which

gives a total of 900 lattice points. I estimate that the slope, m, is . Now,
although there is probably an easier solution, I would try to count the number of
points above the line to see if there are 600 points above the line. The

line separates the area inside the box so that of the are is above
the line.
I find that the number of coordinates with above the line is 30, and the
number of coordinates with above the line is 29. Every time the

line hits a y-value with an integer coordinate, the number of points


above the line decreases by one. I wrote out the sum of 30 terms in hopes of
finding a pattern. I graphed the first couple positive integer x-coordinates, and
found that the sum of the integers above the line

is . The even
integer repeats itself every third term in the sum. I found that the average of each
of the terms is 20, and there are 30 of them which means that exactly 600 above
the line as desired. This give a lower bound because if the slope decreases a
little bit, then the points that the line goes through will be above the line.
To find the upper bound, notice that each point with an integer-valued x-

coordinate is either or above the line. Since the slope through a point is the
y-coordinate divided by the x-coordinate, a shift in the slope will increase the y-
value of the higher x-coordinates. We turn our attention

to which the line intersects

at . The point (30,20) is already counted below the line, and


we can clearly see that if we slowly increase the slope of the line, we will hit the

point (28,19) since (28, ) is closer to the lattice point. The slope of the line

which goes through both the origin and (28,19) is . This gives an

upper bound of .

Taking the upper bound of m and subtracting the lower bound

yields . This is answer .


Diagram

Video Solution , Very Easy


https://youtu.be/PC8fIZzICFg ~hippopotamus1

You might also like